5 votos

Cómo mostrar que esta suma $\sum^{n}_{i=0} \frac{(-1)^i}{n+k+i} \cdot \frac{(m+n+i)!}{i!(n-i)!(m+i)!}=0$

demostrar que :

$$\sum^{n}_{i=0} \frac{(-1)^i}{n+k+i} \cdot \frac{(m+n+i)!}{i!(n-i)!(m+i)!}=0$$ Este problema es de : http://www.artofproblemsolving.com/Forum/viewtopic.php?p=239364&sid=2fbf367cb9fab8240df03e632a41085a#p239364

Difícil cuestión de suma Creo que este problema se han integral de los métodos,Pero no puedo ,Gracias

11voto

psychotik Puntos 171

Voy a suponer que $n+k \geq 1$. (Es decir, $n$ $k$ no son ambos cero simultáneamente.) Deje $p_{n}(x)$ ser el polinomio definido por

$$ p(x) = \displaystyle \sum_{i=0}^{n} \frac{(-1)^{i}}{n+k+i} \frac{(m+n+i)!}{i!(n-i)!(m+i)!} x^{m+i} . $$

Nuestro objetivo es evaluar $p(1)$. Como un primer paso, podemos reescribir $p(x)$

\begin{align*} p(x) &= \sum_{i=0}^{n} \frac{(-1)^{i}}{n+k+i} \frac{1}{i!(n-i)!} \frac{d^{n}}{dx^{n}} x^{m+n+i} \\ &= \frac{d^{n}}{dx^{n}} \left( \frac{1}{n!} \sum_{i=0}^{n} \binom{n}{i} \frac{(-1)^{i}}{n+k+i} x^{m+n+i} \right) . \tag{1} \end{align*}

Mantenga la manipulación de la expresión dentro del corchete de $\text{(1)}$, obtenemos

\begin{align*} \frac{1}{n!} \sum_{i=0}^{n} \binom{n}{i} \frac{(-1)^{i}}{n+k+i} x^{m+n+i} &= \frac{x^{m-k}}{n!} \sum_{i=0}^{n} \binom{n}{i} (-1)^{i} \frac{x^{n+k+i}}{n+k+i} \\ &= \frac{x^{m-k} }{n!} \sum_{i=0}^{n} \binom{n}{i} (-1)^{i} \int_{0}^{x} t^{n+k+i-1} \, dt \\ &= \frac{x^{m-k}}{n!} \int_{0}^{x} t^{n+k-1} (1 - t)^{n} \, dt. \tag{2} \end{align*}

Ahora podemos evaluar la integral de la $\text{(2)}$. Tenga en cuenta que si $m \geq 1$, integración por partes muestra que para cualquier $m, n \geq 0$ la siguiente identidad se tiene:

\begin{align*} \int_{0}^{x} t^{m} (1 - t)^{n} \, dt &= \frac{m!n!}{(m+n+1)!} \left( 1 - \sum_{j=n+1}^{m+n+1} \binom{m+n+1}{j} x^{m+n+1-j} (1-x)^{j} \right). \tag{3} \end{align*}

Desde $n + k \geq 1$, podemos aplicar el $\text{(3)}$ $\text{(2)}$y obtenemos

\begin{align*} \frac{x^{m-k}}{n!} \int_{0}^{x} t^{n+k-1} (1 - t)^{n} \, dt &= \frac{(n+k-1)!}{(2n+k)!} \left( x^{m-k} - \sum_{j=n+1}^{2n+k} \binom{2n+k}{j} (1-x)^{j} x^{2n+m-j} \right) \end{align*}

A continuación, $\text{(1)}$ ahora se reduce a

\begin{align*} p(x) &= \frac{(n+k-1)!}{(2n+k)!} \frac{d^{n}}{dx^{n}} \left( x^{m-k} - \sum_{j=n+1}^{2n+k} \binom{2n+k}{j} (1-x)^{j} x^{2n+m-j} \right) \\ &= \left( \frac{(n+k-1)!}{(2n+k)!} \prod_{j=0}^{n-1} (m-k-j) \right) \, x^{m-k-n} - q(x), \end{align*}

donde $q(x)$ es el polinomio dado por

$$ q(x) = \frac{(n+k-1)!}{(2n+k)!} \sum_{j=n+1}^{2n+k} \binom{2n+k}{j} \frac{d^{n}}{dx^{n}} (1-x)^{j} x^{2n+m-j}. $$

Pero desde $(1-x) \mid \frac{d^{i}}{dx^{i}} (1-x)^{j}$ siempre $i \leq n < j$, se deduce que el $(1-x) \mid q(x)$ y, por tanto,$q(1) = 0$. Por lo tanto, obtenemos

$$ p(1) = \frac{(n+k-1)!}{(2n+k)!} \prod_{j=0}^{n-1} (m-k-j) \underset{m \geq k}{=} \frac{(n+k-1)!(m-k)!}{(2n+k)!(m-k-n)!}. $$

En particular, si $n > m-k$$p(1) = 0$. Claramente, si $ 1 \leq k \leq m \leq n$ $n > m - k$ está satisfecho automáticamente y la conclusión deseada de la siguiente manera como corollay.

4voto

ashish Puntos 320

Tenemos:

$\displaystyle S_{n,m,k}=\sum_{i=0}^n\dfrac{(-1)^i}{n+k+i}\cdot\dfrac{(m+n+i)!}{i!(n-i)!(m+i)!}=\sum_{i=0}^n\dfrac{(-1)^i(m+n+i)...(n+k+i+1). (n+k+i-1)!}{i!(n-i)!(m+i)...(k+i+1).(k+i)!}$

$\displaystyle\quad =\sum_{i=0}^n\dfrac{(m+n+i)^{\underline{m-k}}}{(m+i)^{\underline{m-k}}}\cdot \dfrac{(-1)^i{n\choose i}{n+k+i-1\choose n-1}}{n}$

*Nota: $(x)^{\underline{n}}=x(x-1)...(x-n+1)$

Ahora, Nos caculate diferencias

$\Delta\left[\dfrac{(m+n+i)^{\underline{m-k}}}{(m+i)^{\underline{m-k}}}\right]=\dfrac{(n+k+i+2)...(m+n+i+1)}{(k+i+2)...(m+i+1)}-\dfrac{(n+k+i+1)...(m+n+i)}{(k+i+1)...(m+i)}$

$\quad =n(k-m)\cdot\dfrac{(m+n+i)^{\underline{m-k-1}}}{(m+i+1)^{\underline{m-k+1}}}$

y

$\Delta\left[(-1)^{i-1}.{n-1\choose i-1}{n+k+i-1\choose n}\right]=\dfrac{n+k}{n}(-1)^i.{n\choose i} {n+k+i-1\choose n-1}$

A tal fin por la Sumación por partes, tenemos:

$\displaystyle S_{n,m,k}=\dfrac{1}{n+k}\sum_{i=0}^n \dfrac{(m+n+i)^{\underline{m-k}}}{(m+i)^{\underline{m-k}}}\Delta\left[(-1)^{i-1}.{n-1\choose i-1}{n+k+i-1\choose n}\right]$

$\displaystyle = \dfrac{1}{n+k}\left.\left[\dfrac{(m+n+i)^{\underline{m-k}}}{(m+i)^{\underline{m-k}}}\cdot (-1)^{i-1}.{n-1\choose i-1}{n+k+i-1\choose n}\right]\right|_{i=0}^{n+1}$

$\quad +\dfrac{n(m-k)}{n+k}\sum_{i=0}^n (-1)^{i}.{n-1\choose i}{n+k+i\choose n} \cdot\dfrac{(m+n+i)^{\underline{m-k-1}}}{(m+i+1)^{\underline{m-k+1}}}$

$\displaystyle =\dfrac{(m-k)}{n+k}\sum_{i=0}^{n-1} \dfrac{(-1)^i(m+n+i)!}{(n+k+i+1) i!(n-1-i)!(m+i+1)!}$

$\Rightarrow S_{n,m,k}=\dfrac{m-k}{n+k}S_{n-1,m+1,k+2}=\dfrac{m-k}{n+k}\cdot\dfrac{m-k-1}{n+k+1}S_{n-2,m+2,k+4}=...=$

$=\dfrac{(m-k)(m-k-1)...(m-k-n+1)}{(n+k)(n+k+1)...(2n+k-1)}S_{0,m+n,k+2n}$

dentro de

$S_{0,m+n,k+2n}=\dfrac{(-1)^0 (m+n+0+0)!}{(0+k+2n+0) 0!(0-0)!(m+n+0)!}=\dfrac{1}{2n+k}$

Resultado: $\boxed{\displaystyle\sum_{i=0}^n\dfrac{(-1)^i}{n+k+i}\cdot\dfrac{(m+n+i)!}{i!(n-i)!(m+i)!}=\dfrac{(m-k)^{\underline{n}}}{(n+k)_{n+1}}}$

*Nota: el símbolo de Pochhammer: $(x)_n=x(x+1)...(x+n-1)$

i-Ciencias.com

I-Ciencias es una comunidad de estudiantes y amantes de la ciencia en la que puedes resolver tus problemas y dudas.
Puedes consultar las preguntas de otros usuarios, hacer tus propias preguntas o resolver las de los demás.

Powered by:

X